100th VIDEO❤️ RESOLVED-12 PSEUDO FORCE,ROTATING FRAME PHYSICS,CORIOLIS,CENTRIFUGAL,EULER FORCES

Sdílet
Vložit
  • čas přidán 11. 09. 2024
  • PLEASE SHARE THIS AND PROMOTE THE CHANNEL
    12TH VIDEO OF RESOLVED SERIES FOCUSSES ON THE COMPONENTS OF PSEUDO FORCE :CORIOLIS,CENTRIFUGAL AND EULER FORCES
    "RESOLVED" series will present videos curated on doubts of brilliant -topper batch students with whom I have been involved with different CHAI-NA Hyderabad campuses from 2012 to present. The association with these students was an enriching experience for me💓. I sincerely hope , it will be same for those (teachers and students alike) who will go through these series of videos .
    HERE'S THE RESOLVED SERIES VIDEO ON RELATIVE VELOCITY AND PSEUDO FORCE IN ROTATING FRAME👇
    • RESOLVED-06 !A COMMON ...
    HERE'S THE ELABORATE PICTURE EXPLAINING CYCLONE MOVEMENT DUE CORIOLIS FORCE👇
    drive.google.c...
    RESOLVED series playlist 👇
    • RESOLVED SERIES
    HERE'S THE LINK TO PHYSICS SIR JEE ORIGINALS VIDEO ON PROGRESSIVE STRING WAVE ENERGY ANALYSIS👇
    • ORIGINALS - 🎁STRING WA...
    HERE'S THE PLAYLIST OF PHYSICSSIRJEE ORIGINALS👇
    • PHYSICSSIRJEE ORIGINALS
    HERE'S THE LINK FOR PATHFINDER SOLUTIONS PLAYLIST👇
    • PATHFINDER SOLUTIONS
    OTHER PLAYLISTS
    HERE'S THE LINK FOR "OLYMPIAD WORKOUT SERIES" BOOKS,SYLLABUS,RESOURCES👇
    • ANNOUNCEMENT-2 🧨"OLYMP...
    RESOLVED series playlist 👇
    • RESOLVED SERIES
    HERE'S THE PLAYLIST OF REST OF THE "OLYMPIAD WORKOUT SERIES"👇
    • OLYMPIAD WORKOUT SERIES
    HERE'S PLAYLIST FOR AITS SELECT SOLUTIONS SERIES👍
    • "AITS SELECT" SOLUTION...
    HERE'S THE JEE ADV SOLUTIONS -THE ALPHA AND OMEGA SERIES👇
    • JEE ADV SOLUTIONS - TH...
    HERE'S LINK FOR PLAYLIST OF "JEE MAINS DEBATED QUESTIONS" PLAYLIST👇
    • JEE MAINS DEBATED QUES...
    FOR REST OF THE INTERESTING BRAIN TEASING JEE PHYSICS CHALLENGES AND CONCEPTS , PLEASE SUBSCRIBE TO MY CHANNEL " PHYSICS sir JEE " HERE BELOW
    / channel
    / @physicssirjee
    THE CHANNEL IS AN IMPORTANT RESOURCE TO THOSE STUDENTS PREPARING FOR JEE MAINS, JEE ADVANCED EXAMS IN INDIA AND FOR PHYSICS OLYMPIADS LIKE NBPhO,INPhO,IPHO,HKPhO,BPhO ETC GLOBALLY.
    #PHYSICSSIRJEE,#JEEDAILYLIVE,#JEESPRINT,#JEELECTURES,#JEEPHYSICSLIVE,#OLYMPIADWORKOUT,#RESOLVED,#PSEUDOFORCE,#CORIOLIS,#CENTRIFUGAL,#EULER,#ROTATINGFRAME,
    #jee,#jeemain,#jeephysics,#jeeadvanced,#iitjee,#iit,#iitjeephysics,#jeeadvancedphysics,#AITS,#GMP,#ALLEN,#FIITJEE,#KROTOV,#jeemainsphysics,#neet,#neetphysics,#physicslectures,#schoolphysics,#precollegephysics,#physicsconcepts,#physicschallenges,#liveclasses,#teachinginenglish,#ncert,#irodov,#pathfindersolutions,#pathfinder,#irodovsolutions,#cbse,#ncertphysics

Komentáře • 171

  • @INSP_NITIN
    @INSP_NITIN Před 4 lety +218

    100 vids and 1000 subs , congrats sir your efforts have been phenomenal and both teachers and students will get benefit from such awesome content .

  • @PhysicswithAkashGoyal
    @PhysicswithAkashGoyal Před 4 lety +177

    Congratulations 👏👏👌👍👆

    • @PHYSICSSIRJEE
      @PHYSICSSIRJEE  Před 4 lety +31

      Thank you sir ☺️🙏

    • @rrr1304
      @rrr1304 Před 3 lety +1

      Big fan sir akash sir

    • @rrr1304
      @rrr1304 Před 3 lety

      @@PHYSICSSIRJEE u too sir.

    • @sushantsrivastava7013
      @sushantsrivastava7013 Před 3 lety +5

      Big Fan Of You sir ....... Teachers like You, Nitin Sir , Janardhan Sir , Ashish Arora sir , Manas Patnaik Sir have made me Love Physics ........ Thanks A LOT

    • @vedant6598
      @vedant6598 Před 3 lety +2

      Two hidden legends of physics on you tube Akash sir and Janardhan sir ❤️❤️

  • @roam2explore588
    @roam2explore588 Před 4 lety +41

    No of Subscribers on CZcams vs Quality of content follows Maxwellian distribution

  • @varuduhero606
    @varuduhero606 Před 4 lety +32

    nitin sir and janardhan sir you both have made me love physics, the concepts you teach are really very interesting😊, you make tough questions a piece of cake. Thanks you sir and congrats for both of you sir for reaching 1K subs🥳

  • @deevyanshkhadria9274
    @deevyanshkhadria9274 Před 3 lety +13

    i find u are better than physics galaxy sir. you are great. if i see your vedios i think it is a golden mark on my jee prepration

    • @PHYSICSSIRJEE
      @PHYSICSSIRJEE  Před 3 lety +45

      Thanks for the compliment 🙏 🙂and you are entitled to have your opinion . Everyone does but I generally don't encourage comparisons among teachers especially on public platform. Everyone has their own style but with a common goal of trying their best to make students understand.
      Ashish Arora sir has done a lot of work in this field over a long period of time . It would be great for students in our country if a lot of upcoming teachers can emulate his hardwork.
      Thanks again and keep following the channel 😊

  • @sohansingh2022
    @sohansingh2022 Před 10 měsíci +3

    Man its a different feeling watching it from IIT Kanpur Hall 5!! Emotional.... Anyone who is watching this video, you will have a good time in JEE Advanced

  • @Meghana_Nallamilli
    @Meghana_Nallamilli Před 4 lety +9

    Sir, you are definitely not overestimating yourself. The content delivered is indeed unique, enriching in concepts and top-notch. The reason why views are maybe not as expected is probably because mains is approaching for my friends. Congratulations for the 100th video and 1k subscribers!! Hope you will have many more viewers, I will continue to support this channel even after my JEE being a physics lover.

  • @_alpinist535
    @_alpinist535 Před 2 lety +5

    thankyou sir for providing this quality content and teaching it with your dedicated efforts .
    i was confused in this concept {basically i had doubt in what was given in hcv}
    i cannot pay your "guru dakshyana" through this youtube channel but the only gift i can give you is that your effort in this lecture has given me a new weapon for my jee advanced which i am confident will definitely reflect in my jee advanced score(because i understood the concept taught in this video well and also analysed the cyclone movement image that you gave in pdf)
    congratulation for 14k subs

    • @PHYSICSSIRJEE
      @PHYSICSSIRJEE  Před 2 lety

      Thanks a lot for the encouragement ☺️. I wish the content gets appreciated by more people like you 🙏

  • @nimitjain8163
    @nimitjain8163 Před 2 lety +2

    This video cleared many misconceptions. Until now I used to think, pseudo force= centrifugal force. Thanks sir for making such videos

  • @sekhar9000
    @sekhar9000 Před 4 lety +3

    i have seen a very high hard work in changing colors of font to make student feel comfortable in understanding,and sir you did your level really the best.

  • @ManishSingh-jz2hx
    @ManishSingh-jz2hx Před 2 lety +1

    This video is really in a league of its own. The fundamentals of rotation pseudo forces have been explained in such detail that everything can be understood by a normal student. You sir are a gem of a physics teacher. Many can understand what you are trying to teach, but only a few can make it understand in such an easy manner.

  • @lakshitasharma4709
    @lakshitasharma4709 Před 3 lety +4

    congratulations sir!! thnk u for ur efforts u are a true legend; my physics improved very much bcoz of this beautiful channel i can't describe in words❤❤❤❤💚💚💙🤩🤩

    • @ridhi8333
      @ridhi8333 Před rokem

      Di abhi aap konse college me h?

    • @ridhi8333
      @ridhi8333 Před rokem +1

      @@Nishantpaswan456 op 🔥🔥

  • @sumithasuresh4597
    @sumithasuresh4597 Před 2 lety +1

    I pray for your long-life sir. Such beautiful, top-notch content being delivered with such passion (I could fully feel it in your voice) for free on this platform is simply unbelievable. Thankyou so very much sir

  • @Verylastchangs_older_brother

    Thank u sir ❤️. Aapki help ki wajah se mera physics me interest drastically increase hua hai . Thnks from bottom of my heart.

  • @physicsisdope4892
    @physicsisdope4892 Před 3 lety +7

    Done #100
    April 9
    This lecture was pure masterpiece 👌🏻

  • @shubhamraj4879
    @shubhamraj4879 Před 3 lety +1

    Amazing video sir.
    Sir main problem ye hai ki JEEA ke itne jyada competition ke kaarn students aapke channel ko jyada share nhi krte, your content is like gold which they want to keep it to themselves only, kyoki agr sbko pta chl jayega to sb is level pr aa jayenge, aur baki jo channels famous hai wo isiliye hai kyonki sbko pta ki unka content dekhne nhi dekhne se JEEA ke prepration pr koi frk nhi padega but ye dekhne walo ko surely upper hand rahega, Even mere dosto ko aapke channel ke bare me pta tha pr unhone kabhi nhi bataya.

  • @mihirpandey8372
    @mihirpandey8372 Před 2 lety +1

    The explanation was insane ,It was very helpful .Thank you sir

  • @ANURAGGUPTA-fe4ek
    @ANURAGGUPTA-fe4ek Před 3 lety +2

    Thanks to the cylindrical coordinate system the homework problem you gave becomes easy and yes result is even true for general case when r2 and r1 are not collinear ......

  • @amiyancandol4499
    @amiyancandol4499 Před 3 lety +2

    Back from 200 sir 🥳🥳🥳 congrats

  • @hemantgupta1257
    @hemantgupta1257 Před 3 lety +2

    No body can give this type of content in free

  • @mystic3549
    @mystic3549 Před rokem +2

    Aag laga di !!❤️❤️ Never would I have visualised it without this vid

  • @manassingh1710
    @manassingh1710 Před 3 lety +2

    U r op sir........

  • @advaitrisbud2510
    @advaitrisbud2510 Před 4 lety +2

    Congratulations 🎉 1000th subscriber soon!

  • @Algoner
    @Algoner Před 4 lety +1

    Congratulations🎉🎉🎉 sir for 1k subscribers and and I wish u have 1M subscribers and more in few days

  • @korigamik
    @korigamik Před 3 lety +1

    This was the best video so far sir. I am grateful.

  • @temporary9508
    @temporary9508 Před 2 lety

    Sir you are too good for indian studs.. That's something I truly feel from my heart

  • @atridevbhattacharya5050
    @atridevbhattacharya5050 Před 2 lety +1

    sir loved your level of teaching and the content and the way of teaching

  • @Sunny4072
    @Sunny4072 Před 2 lety +1

    very nice video ............ got the same result sir thank u

  • @SAURABHANANDLOVE
    @SAURABHANANDLOVE Před rokem +2

    First of all thanks for such an explanation, really an eye opener concept...always misunderstood by most of us....
    I have one doubt sir...while proving the theorem in mathematics part (time 8:27 to 10:23 ) you have used a subscript with Euler part of angular acceleration, Alpha of (A) i.e; angular acceleration of A ......I think angular acceleration (alpha) used in Euler part will be of rotating frame but not of the object being observed...
    Am I right sir or I have made mistake somewhere ?

    • @PHYSICSSIRJEE
      @PHYSICSSIRJEE  Před rokem

      Yes , you are right 👍🏼🙂. That was typo at 10:00 on my part. In the previous step , I just wrote alpha without subscript. But from next step, upon multiplication of mA term , somehow, I wrote alpha A , and kept carrying it in further steps without any use. Yes alpha and Omega are of the rotating frame itself 🙂👍🏼. Thank you for pointing out 🙏.

    • @SAURABHANANDLOVE
      @SAURABHANANDLOVE Před rokem

      @@PHYSICSSIRJEE I am your big admirer...by posting conceptual videos on youtube you have helped so many students and knowledge seekers who cannot contact you and take guidance from you in personal. Please don't stop posting conceptual videos...honest request... 🙏

  • @simisimi9362
    @simisimi9362 Před rokem

    This is the best video to explain coriolis force.👍💯thank you sir 🙌🙌

  • @arunayarramreddy4235
    @arunayarramreddy4235 Před 4 lety +1

    Great choice for 100th video sir..

  • @simplephysics104
    @simplephysics104 Před 2 lety

    Thank you very much sir...I learnt this rarer concept by viewing this video second time...

  • @grandson_f_phixis9480
    @grandson_f_phixis9480 Před 2 lety

    Thank you very much sir🥳🥳🥳🥳

  • @anusrt73
    @anusrt73 Před 2 lety

    This is just beautiful, it is a masterpiece

  • @kingpatil2882
    @kingpatil2882 Před 4 lety +3

    *Worth*

  • @samriddhimishra557
    @samriddhimishra557 Před 4 lety +2

    sir your content is really great!!!!!!!

    • @samriddhimishra557
      @samriddhimishra557 Před 4 lety

      Sir you said you would take up some challenging ques from EMI PATHFINDER please sir do take them,A humble request

  • @harshithaharshitha9845
    @harshithaharshitha9845 Před 4 lety +1

    Congratulations sir

  • @giriganta9019
    @giriganta9019 Před 3 lety

    Congratulations sir....I learned proper physics... thank you sir

  • @tanuj8459
    @tanuj8459 Před 2 lety +1

    Wow

  • @SandeepKumar-nn6pr
    @SandeepKumar-nn6pr Před 2 lety

    Before watching your video, I had used this concept in a problem.
    Many people forget to take the effect of rotating frame in the relative motion and establish erroneous equations and thereby find faluty answers with pseudo satisfaction of doing that way.
    After watching this video, I am excited to see that atleast someone has mentioned the same concept in a very simple manner.

  • @subhroHalder427
    @subhroHalder427 Před 2 lety

    Thank you sir, amazing concept,mind blowing 🎐✨

  • @AK56fire
    @AK56fire Před rokem

    In Kleppner and Kolenkow's book: An Introduction to Mechanics, on page 34 on the topic titled "Acceleration in Polar coordinates", it has been mentioned that: "when r and theta both change, then Coriolis acceleration acts which is "real" and is "In contrast" to the Coriolis force which acts in a rotating frame of reference. "
    What I'm trying to understand is that:
    (1) If we are analyzing the situation using polar coordinate system, then if r and theta both are changing, then Coriolis acceleration that acts (which by the way is real according to Kleppner and Kolenkow), is this Coriolis acceleration different from the Coriolis acceleration that would come into play due to Coriolis force, if we analyze the same situation from a rotating frame of reference ?
    (2) Can Coriolis acceleration (real one) and Coriolis acceleration (due to Coriolis force when seen from rotating frame of reference) act simultaneously at a moving body(whose r and theta both are changing with time), if we observe from a rotating frame of reference using polar coordinate system ?
    I request you to kindly make a video explaining this, or atleast write your answer as comment here..

    • @redsurfer_255
      @redsurfer_255 Před rokem

      dude read the book more clearly , in the book it says that in rotating frame(which perhaps isnt our reference frame or laboratory frame) coriolis force acting isnt real but in the kinematics section .. the paragraph you read particle has some speed , accn , initial position etc , so we are interpreting the equation of motion in polar coordinates that is .. for some accn in cartesian coordinate we break the components of accn in polar coordinates and we get the expression as shown in the video for example 2w*r is the accn in tangential or say transverse direction given that omega is constant. in the book ,we are just interpreting motion of particle in polar coordinates. now in circular motion problems , we consider that the body is moving in some way in our laboratory system , but in the frame of the rotating body the other body let say some other omega relative to itself and then we use the help of polar coordinates to find that relative accn or velocity.hence in the first case the sh*t is real to us but in second case the sh*t is pseudo

    • @AK56fire
      @AK56fire Před rokem

      @@redsurfer_255 I've read your reply multiple times. However, it still isn't clear to me. Could you kindly elaborate pointwise on the questions in my original post.

  • @krishnavenia3899
    @krishnavenia3899 Před 4 lety +2

    Sir miru thopu

  • @vineetjain3172
    @vineetjain3172 Před 2 lety

    Best video learnt a lot of concepts

  • @sekhar9000
    @sekhar9000 Před 4 lety

    excellent sir,keep going,all the best sir jee

  • @anishtakshak6515
    @anishtakshak6515 Před 3 měsíci

    Thank you sirji😊❤

  • @prathamrana8916
    @prathamrana8916 Před 2 lety

    Thanks a lot sir ,
    All doubts cleared .❤️

  • @Ayush_JEE24
    @Ayush_JEE24 Před měsícem

    Currently in IITP
    still watching this for my revision :)

  • @priyanshrawat442
    @priyanshrawat442 Před rokem

    Mind blowing

  • @ponnapu
    @ponnapu Před 4 lety

    congo sir really sir very happy for you congrats sir

    • @ponnapu
      @ponnapu Před 4 lety

      i can see a lot of effort behind this video yet it would be a delight for any physics lover to watch it truly loved the illustrations

  • @arvindpatil9394
    @arvindpatil9394 Před 3 lety

    Only 3 physics channels are trending now.
    INSP ,physics sir jee and physics with akash goyal❤️❤️❤️❤️❤️

  • @Letscelebratescience
    @Letscelebratescience Před 10 měsíci

    Thank you sir for your amazing content

  • @tanveeersharma4334
    @tanveeersharma4334 Před 4 lety +2

    1000 subscribers soon sir!!
    it feels good to be here from the beginning ✌🏻

  • @physicsgram7127
    @physicsgram7127 Před 3 lety

    U r doing my best wishes

  • @dishankchoudhury4551
    @dishankchoudhury4551 Před 2 lety

    Wow sir maja aa gya pura

  • @thunderstorm847
    @thunderstorm847 Před 4 lety

    Congratulations for 1K subscribers sir 🎉🔥

  • @yashagrawal8592
    @yashagrawal8592 Před 3 lety +2

    Sir if the rotating table is itself translating with velocity v , Can we still use the rotating frame in same manner or some other pseudo forces will appear ?

  • @srisurya9836
    @srisurya9836 Před 3 lety

    Love from Andhra sir❤️❤️

  • @mythravarunbojanapally4226

    Well sir

  • @modernphysics7474
    @modernphysics7474 Před 3 lety

    Awesome explaination sir 👍👍 🔥🔥 ......love from Chandigarh 🙂

  • @yashaswini7352
    @yashaswini7352 Před 4 lety +1

    Congratulations sir !! This channel deserves to be seen by EVERY physics lover , physics is such a beautiful subject and with you it becomes even more beautiful !! 😊

  • @endlesseducation2845
    @endlesseducation2845 Před 3 lety

    #75 For My Revision( This video deserves a gold 🏅)

  • @harshdixit301
    @harshdixit301 Před 3 lety +2

    Is this coriolis force and Euler force application on jee advanced sylaabus

  • @CallingLife
    @CallingLife Před 6 měsíci +1

    sir in euler force the alpha should be of obsever b i think

  • @brocklesner2457
    @brocklesner2457 Před 3 lety +1

    KIndly explain the difference between coroilis and centrifugal force physically since both are due to rotation effects i.e. also give examples when is coriolis present and centrifugal absent or vice versa, if any.
    Also do a full series on topics from basic to advanced level on topic like newtons laws, constraint motion and rotational motion which are the toughest topics.

    • @PHYSICSSIRJEE
      @PHYSICSSIRJEE  Před 3 lety +1

      The points in first paragraph of your comment is completely covered in the long video. Some examples are provided and some more , I will do in a future video
      Coming to the second paragraph of your comment , it's a very good suggestion 👍🙂. I will definitely take it up in near future. please kindly stay tuned 🙏

  • @as.if_0077
    @as.if_0077 Před 3 lety

    Thank you so much sir

  • @aditidhingra4256
    @aditidhingra4256 Před 3 lety +1

    😊👏

  • @arpanhalder7803
    @arpanhalder7803 Před 3 lety +2

    What a nice video sir!
    Sir I have a doubt if I may ask, shall be really thankful if you would clarify in ur free time.
    What exactly is relative angular velocity if two objects are moving on concentric circles? Is it (w1-w2) ?
    or is it (V1-V2)/ (r1-r2) ? {r1,r2 being position vectors}
    Two of them are giving different answers and a few searches on internet has further confused me a lot.

    • @PHYSICSSIRJEE
      @PHYSICSSIRJEE  Před 3 lety

      Neither if two objects are points, it's (Vrel perpendicular to Rrel)/Rrel

    • @arpanhalder7803
      @arpanhalder7803 Před 3 lety

      @@PHYSICSSIRJEE Ok sir, actually second case was this only , could not frame here in comment correctly.
      Thanks a lot sir, I got the clarity now.

    • @kavi5990
      @kavi5990 Před rokem

      @@PHYSICSSIRJEE Sir here as points don't rotate u have neglected the Wa term
      But if it's two people then will it be[ Vrel trans -Wa(distance between them) ]/perpendicular distance between them
      Here I am initially taking the case when their velocities are parallel
      And Wa refers angular velocity of frame of reference particle
      Also what if it's mentioned as particles do we consider them to be points or people

  • @borutouchiha1942
    @borutouchiha1942 Před 3 lety

    Sir how to use your channel as a dropper I am enrolled in a coaching and preparing for jee 2022 from scratch and I watched one of your videos and I was shocked how clearly and greatly explain tough concept 😁

  • @invisiblestring5885
    @invisiblestring5885 Před rokem

    thank you sir

  • @TrishaBafna-en8vq
    @TrishaBafna-en8vq Před 11 měsíci

    Sir @20:00 can u pls tell how centrifugal force is in horizontal direction
    i am confused coz in formula, it was -ma wx(wxrab) but rab is not horizontal since the object is behind the line of sight of obeserver
    then how horzontal force can act

  • @svsharma3893
    @svsharma3893 Před 4 lety

    Also sir jee I am curious to know your daily routine

  • @mohithmvs5097
    @mohithmvs5097 Před 4 lety

    The same concept when I read in David Morin introduction to mechanics there are many things which are misunderstood but now I understood very well thank you sir love from south india

  • @sushantsrivastava7013
    @sushantsrivastava7013 Před 3 lety

    Concept of Coriolis Force ...... I dont know why give me a little dizziness

  • @aaryangupta2328
    @aaryangupta2328 Před rokem

    Sir pls clear some of my doubts regarding this topic :
    1) What is the difference between a translatory frame and rotatory frame can we not have perpendicular velocities in a translatory frame if yes then our line of sight is rotating itself doesn’t it become a rotatory frame ..? How do we differentiate bw the two I am not able to understand this.. does translatory means we can only have velocities along the line of sight .? And lastly if my line of sight is rotating and I’m myself rotating does this means same. .? I need to clear these doubts then only I’ll be able to understand coriolis and other pseudo forces

    • @PHYSICSSIRJEE
      @PHYSICSSIRJEE  Před rokem +2

      You can watch the previous resolved video mentioned (check in description) at the start to understand the difference between translating and Rotating frames...
      If doubt persists you can DM me in my app, I can explain using audio message when I am free, instead of typing message here🙂

  • @amitguptacheers
    @amitguptacheers Před 3 lety

    Sir in your theory will observer in frame B (rotating frame ) observes path of particle A to be straight line along vector rab.

  • @sayanjitb
    @sayanjitb Před 3 lety +1

    So for the last scenario, we saw that stationary person B seemed to be rotating in radius r_2 circle in the frame of the rotating observer. But my question is when both come at a minimum distance, i.e. (r_2 -r_1), then how is it possible for a rotating observer to measure this distance as r_2? Please help me here!

    • @PHYSICSSIRJEE
      @PHYSICSSIRJEE  Před 3 lety

      At 26:00 , the OBSERVER A doesn't see B to be rotating in circle with "A" as center . He observes him to be in circle of radius R2 with center at centre of turntable. It's like observing a circular motion from an off-center Point

  • @twistedargument4308
    @twistedargument4308 Před 3 lety +1

    Sir if a ball is rotating around a string then we say that acceleration of ball is mrw² outward w.r t rotating frame but according to theory discussed in video acceleration of ball w.r.t to rotating body should be F real+F pseudo=-mrw²+mrw²(centrifugal force due to rotation of the centre.then sir why the earlier one is wrong.i am pretty confused.

    • @twistedargument4308
      @twistedargument4308 Před 3 lety +1

      Sir please clear the doubt I am pretty confused

    • @PHYSICSSIRJEE
      @PHYSICSSIRJEE  Před 3 lety

      Centrifugal depends on where is the rotating frame in your example on the string

    • @twistedargument4308
      @twistedargument4308 Před 3 lety

      @@PHYSICSSIRJEE sir I am taking centre of the circle as the rotating frame

    • @PHYSICSSIRJEE
      @PHYSICSSIRJEE  Před 3 lety

      @@twistedargument4308 then centrifugal force is mrw^2 and linear pseudo force term is zero.
      But , if observer is at midpoint , then centrifugal force and linear pseudo force term each are mrw^2/2 but total pseudo force is still mrw^2.
      Pick your observer rigidly attached to any point, the centrifugal and linear pseudo force term individually Change but total pseudo force is Same

    • @twistedargument4308
      @twistedargument4308 Před 3 lety

      @@PHYSICSSIRJEE yes sir ,if body is rotating around centre then w.r.t to centre fnet=0(f real+f pseudo),n??

  • @rishabhshetty4304
    @rishabhshetty4304 Před 3 lety +2

    At 7:25 I have difficulty in understanding the 3rd step .... we wrote differentiation of (Vab)r , of the 2 terms the first one should be differentiation of rotating vector (Vab)r in rotating frame , however is differentiation of (Vab)r and (Vab)t same ? I know that there will be difference in second term involving omega however would the first term be same . I also had doubt in visualizing (Vab)r as a rotating vector , is it like a tangent going around a circle with centre at observer ?

    • @rjaph842
      @rjaph842 Před 3 lety

      Yeah nice question bro,is differentiation of Va/b in Translation same as in rotating frame i.e Va/b(rotating frame)

    • @rishabhshetty4304
      @rishabhshetty4304 Před 3 lety

      @@rjaph842 Do you know the reason ?

    • @rjaph842
      @rjaph842 Před 3 lety

      @@rishabhshetty4304 i also can't understand why he did that,i was hoping he can reply

    • @rishabhshetty4304
      @rishabhshetty4304 Před 3 lety

      @@rjaph842 Ok , Join discord you may ask there .

    • @rjaph842
      @rjaph842 Před 3 lety

      @@rishabhshetty4304 I don't get, what's discord and how do i join?

  • @mishthiagarwal1680
    @mishthiagarwal1680 Před rokem

    Sir at 28:59 the turn table is moving anticlockwise with omega so you considerd omega to B as outwards. Sir i didnt understand this pls can you explain.

  • @bhuvneshmathur8007
    @bhuvneshmathur8007 Před 3 lety

    Sir (20:05), you have mentioned that the relative velocity is away from from observer in rotating frame. You have taken vrel as horizontal. Could you please explain it. Why vrel is nt the vector difference between two velocity

    • @PHYSICSSIRJEE
      @PHYSICSSIRJEE  Před 3 lety

      Explained in the previous video as mentioned . Link in description

  • @jitendrapandey1085
    @jitendrapandey1085 Před 3 lety

    Sir aap unaccademy pr aaye ge...!..

  • @mihir6096
    @mihir6096 Před 3 lety

    sir in second illustration (21:00) sir waha total force wrt to rotating observer me euler force ke alwa sab force ayegena???..........,,,,,,, And sir centripetal force jab particle circuler motion karta he tab ham toward center lagate....so ese centrifugal force kab kab lagna hota he...jese coriolis force tab lagate he jab rotating observer and particle ke bich relative motion ho vese centrifugal kab lagate he?.......ya fir rotating observe ke fram me always centrifugal force lagna hota he?
    🙏🙏

    • @PHYSICSSIRJEE
      @PHYSICSSIRJEE  Před 3 lety +1

      Answer to first line of your comment : Yes ✅
      Answer to the last line of your comment : Yes✅

    • @mihir6096
      @mihir6096 Před 3 lety

      @@PHYSICSSIRJEE ok thank you sir 🙂🙂

  • @abhinavbhatt7952
    @abhinavbhatt7952 Před 3 lety

    Sir in iroovar cone rotation problem you had mentioned da/dt=a x w but here we have done as da/dt = a x w + the normal term we would get by differentiating a where a is rotating vector

    • @PHYSICSSIRJEE
      @PHYSICSSIRJEE  Před 3 lety +1

      In the irodov cone problem , I did not consider a Rotating frame attached to the axis . In this video , during differentiation , I mentioned it was about a Rotating frame

    • @abhinavbhatt7952
      @abhinavbhatt7952 Před 3 lety

      sorry sir for not being attentive and being silly. Thank you for clarifying@@PHYSICSSIRJEE

  • @kshitizrana5514
    @kshitizrana5514 Před 3 lety

    Cant explain how frustated i was i thought of this frame at my own but was confused that if we dont consider that rotational factor in relative velocity how can we get a fx ,it was literally amazing sir

  • @aaryangupta2328
    @aaryangupta2328 Před rokem

    Sir pls clear this small doubt at 5:47 how did u write rate of change of rAB as VA-Vb as length would only change if the velocities are along r vector suppose if velocities are perpendicular to position vector then dr/dt shells be zero but it gives a value how sir?

    • @PHYSICSSIRJEE
      @PHYSICSSIRJEE  Před rokem +1

      It's a good doubt 🙂. What you said is right if it was rate of change of MAGNITUDE of rAB , but it was rate of change of vector rAB , then net velocity vectors should be considered and not just the components along rAB

    • @aaryangupta2328
      @aaryangupta2328 Před rokem

      @@PHYSICSSIRJEE thnku so much sir ❤️

  • @aniketkini9255
    @aniketkini9255 Před 3 lety

    Sir hoping for a Collab soon with nitin sir

  • @rjaph842
    @rjaph842 Před 3 lety

    Help here sir,in 7:25,you are taking differentiation of Va/b in rotating frame to be equivalent to differentiation of Va/b in translating term, kindly shed light there sir

    • @PHYSICSSIRJEE
      @PHYSICSSIRJEE  Před 3 lety

      There are two terms after differentiation in performing d/dt of Va,b in rotating frame.

  • @ss0047
    @ss0047 Před 3 lety

    sir one help sir.., can u please solve Q 1.216 in irodov sir .., I'm really confused with the concept involved sir..,
    Please sir .., please

    • @PHYSICSSIRJEE
      @PHYSICSSIRJEE  Před 3 lety

      Check step -1 of this Solution. It's on gravitational pressure 👇
      czcams.com/video/iaZr0UycNGM/video.html

  • @fanofjanardhanudusir4848

    Sir i hope you can be eligible for monitisation please do it sir you will get upto 1,64,235.7988 ruppes

  • @thermmech4835
    @thermmech4835 Před 3 lety

    sir but w(omega) wrt boy of that another outside boy will be downside so why you directed it upwrds@29:15

    • @PHYSICSSIRJEE
      @PHYSICSSIRJEE  Před 3 lety

      I didn't mark Omega , I marked Vrel .
      Eyes of A in the figure are ROTATING anti-clockwise , so he will see B moving with the combination of red and green velocity components marked in diagram.
      I suggest you to please go through the previous Resolved VIDEO to understand relative velocity in ROTATING frame. It was mentioned at start of this video. Link is in description.

    • @thermmech4835
      @thermmech4835 Před 3 lety

      @@PHYSICSSIRJEE sir i mean into the plane by mistake I wrote it downside I am talking about omega only that it will be inside the plane not coming out of the plane

    • @PHYSICSSIRJEE
      @PHYSICSSIRJEE  Před 3 lety

      @@thermmech4835 anticlockwise Omega is out of the plane , right?

    • @thermmech4835
      @thermmech4835 Před 3 lety

      @@PHYSICSSIRJEE sir but out of plane is for first rotating object but if see for other object(B) it will have inward i mean "into the plane" omega

    • @PHYSICSSIRJEE
      @PHYSICSSIRJEE  Před 3 lety

      @@thermmech4835 relative Omega doesn't work like relative v . It's a pseudo vector. Also relative Omega was not written in the solution. That's why, please Watch, the previous video to this Completely as mentioned in the description to get better clarity.
      All the best 👍🙂

  • @PrabhatKumar-rx6dl
    @PrabhatKumar-rx6dl Před 4 lety +1

    ohh itna types ka pseudo forces hota h kya.... in 8 yrs of teaching i knew that transaltional and coriolisi only

  • @svsharma3893
    @svsharma3893 Před 4 lety

    I was the hundrenth person to like it
    Haha

  • @ponnapu
    @ponnapu Před 4 lety

    janardhan sir can u plz tell me the method to find the depth of vertex of a hanging catenary(question given in pathfinder nlm)

    • @ponnapu
      @ponnapu Před 3 lety

      @Puspal Paul yaa thank u for telling me i have actually seen it i posted this doubt before sir uploaded that video

  • @atharvakumar9808
    @atharvakumar9808 Před 4 lety +1

    1000 subscriber soon sir ..
    We love you sir. 😊

  • @mr.rachetphilanthrophist601

    Wow